Convergence and divergence of a series

Click For Summary
The discussion focuses on determining the convergence or divergence of a series using the ratio test and L'Hôpital's rule. Participants suggest simplifying the expression using elementary algebra and caution against relying solely on the ratio test, as it may not always be applicable. There is an emphasis on considering the series as a sum of two parts, where the convergence of each part can influence the overall convergence. The conversation also highlights the importance of absolute convergence in assessing the behavior of the series. Ultimately, the participants encourage careful analysis of the problem before proceeding with convergence tests.
smart_worker
Messages
131
Reaction score
1
B]1. Homework Statement [/B]
Find whether the series is convergent or divergent

Homework Equations



lim.JPG


The Attempt at a Solution



By ratio test I have,
limit.JPG


I would apply L'Hôpital's rule to find the value of limit but before that how do i simplify the expression? It has fractional part both in the numerator as well as in the denominator.
 
Last edited:
Physics news on Phys.org
smart_worker said:
B]1. Homework Statement [/B]
Find whether the series is convergent or divergent

Homework Equations



View attachment 74254

The Attempt at a Solution



By ratio test I have,
View attachment 74255

I would apply L'Hôpital's rule to find the value of limit but before that how do i simplify the expression? It has fractional part both in the numerator as well as in the denominator.

Just use elementary high-school algebra. Alternatively, look more carefully at the problem before even starting. Maybe the ratio test won't work; there are times when it doesn't.
 
Consider breaking it into two sums...the sum of convergent series is convergent, however if one diverges, the sum of the two diverges (generally).
 
If you want to simplify the large fraction in
<br /> \lim_{n\to \infty} \left(\dfrac{\left(\dfrac{(n+1)^2}{2^{n+1}} + \dfrac{1}{(n+1)^2}\right)}{\dfrac{n^2}{2^n} + \dfrac 1 {n^2}}\right)<br />

treat it the way you would a complex fraction. As has been stated above, however, I'm not sure this approach will generate a positive result.

Think about the idea that if both \sum_{i=1}^\infty a_n and \sum_{i=1}^\infty b_n are absolutely convergent then
\sum_{i=1}^\infty \left(a_n + b_n \right) is absolutely convergent.
 
RUber said:
Consider breaking it into two sums...the sum of convergent series is convergent, however if one diverges, the sum of the two diverges (generally).
Not sure what you mean by "generally" other than perhaps it means "sometimes" because you know it's false in general.
 
  • Like
Likes RUber
Question: A clock's minute hand has length 4 and its hour hand has length 3. What is the distance between the tips at the moment when it is increasing most rapidly?(Putnam Exam Question) Answer: Making assumption that both the hands moves at constant angular velocities, the answer is ## \sqrt{7} .## But don't you think this assumption is somewhat doubtful and wrong?

Similar threads

  • · Replies 4 ·
Replies
4
Views
1K
  • · Replies 2 ·
Replies
2
Views
2K
  • · Replies 3 ·
Replies
3
Views
1K
  • · Replies 5 ·
Replies
5
Views
2K
  • · Replies 4 ·
Replies
4
Views
2K
  • · Replies 8 ·
Replies
8
Views
2K
  • · Replies 4 ·
Replies
4
Views
1K
  • · Replies 11 ·
Replies
11
Views
3K
  • · Replies 4 ·
Replies
4
Views
2K
  • · Replies 6 ·
Replies
6
Views
2K